Halting Problem Discussions

Components:

Potential New Pages:


Part 1 Halting Problem


The HaltingProblem states that it is impossible to write a program that runs in finite time that is able to decide whether an arbitrary program will halt (with a certain input).

The HaltingProblem does not state that it is impossible to decide for a specific program whether it is going to halt. It is even possible to write programs who make this decision. They simply work only on a subset of all programs.

To be correct about terms, the HaltingProblem is actually the question, "Given this code X, will it terminate (halt)?". Usually the only reason to ask this formulated in such a way is to give an example of a problem for which there is no recursive enumerable algorithm, i.e. an algorithm that will itself always produce a result.

See Also: WhatDoesHaltingMean


Here is the proof of the HaltingProblem:

WhatDoesHaltingMean?


That argument applies only to a TuringMachine or equivalent. A real machine has a finite amount of state, so you can just run the program in question until it either halts or repeats a state. The machine P runs on would have to be larger than the one Q runs on (a TortoiseAndHare approach would have P's machine large enough to contain two copies of Q's machine state), so Q cannot fully simulate P.

Quite so. But the general HaltingProblem isn't interesting because it indicates a limit to what debuggers do. It's interesting because (a) it's demonstrably insoluble and (b) it's easy to transform large classes of problems into it. This is why it's one of the fundamentals of computability.


Are there any known interesting problems to which this theorem is applicable?

Yes. The answer to the question, "Can I please have 3 million dollars for my research project to detect by inspection whether a program will halt?" is no. Surprising useful. Just as is, "Ha Hahahaha, My computer's instruction set is more powerful than yours!!" being a nonsense claim, thanks to establishing quickly that they are all TuringMachine equivalent. Saved decades of advertising one-up-manship.

Actually, the ChurchTuringThesis hasn't been proved and isn't proved by alluding to the general HaltingProblem. It just hasn't been disproved. Cf. the ScientificMethod.

However, that doesn't affect the fact that proving a computer's instruction set Turing-equivalent demonstrates that it is no more powerful than a Turing machine.


Known interesting problems... yes. "Will this line of code ever be executed?" If we replace the line of code with the HALT instruction, we can see the question is equivalent to the halting problem. (Hence perfect optimization is impossible.)

"Does this program do the same thing as this other program?" If we choose as our second program the one-liner "HALT" (which always halts), we can see this question is, in general, equivalent to the halting problem.

The theorem shows there is a limit to what you can do with a static type checker. If you make a static type system too powerful, it becomes UniversalTuringMachine-capable, and hence you may have to solve the halting problem before you can say whether a given program is type-safe. (Avoiding this is essentially what the word "static" means here.) This is why designing a good static type system is hard, and part of why so many of us prefer dynamically checked languages like Smalltalk.

There are lots of more-or-less trivial variations. I was recently asked to give a bound on the stack size for an arbitrary program. Clearly we can construct a program which halts if and only if a given program exceeds such and such a bound, so finding the bound is equivalent to solving the halting problem.

In a way, this theorem is saying there are no short cuts. Turing earlier showed that a UniversalTuringMachine can tell you what any other TuringMachine will do by emulating it, which is a powerful positive result, but the halting theorem says that emulating it is the only way, which is a powerful negative result. Turing gave with one hand and took away with the other.

You can think of many (perhaps all) physical processes as being computations, with inputs, outputs and evolving states. A UniversalTuringMachine is rather a simple machine, therefore we might expect many physical processes to be UniversalTuringMachines if you look at them right. Hence predicting their behaviour by way of short cuts like partial differential equations is not going to work. Not only is computer-modelling a respectable approach to physics, it may ultimately be the only respectable approach. -- DaveHarris

But... It's also a good excuse for giving up too early. For example, if you want to know the maximum stack depth of a program, and find that there are no recursive loops of calls in the program, then finding the maximum depth is really quite doable. However, if there's a loop in the static "call tree" of the program, then you're back to the halting problem - and can't be sure how much stack will be needed. -- JeffGrigg

Unless you happen to be able to prove some relevant property of the specific algorithm it implements, and prove that it does implement that algorithm, of course. I hesitate to make this comment here, because it tends to confuse the student more than it clarifies things, prompting thoughts such as "What about a program which will either prove that a TuringMachine halts, or prove that it is impossible to do so for that specific TuringMachine?" (which are answered by other theorems...), but I trust that anybody who considers this seriously harmful will delete it. Wiki is fun that way. -- DanielKnapp


Another important consequence of the halting problem is being unable to know whether a given mathematical formula is a theorem or not (given a set of axioms and inference rules). It is possible to construct an effective enumeration of all theorems: start with the axioms, then the set of all formulas that can be deduced from combinations of axioms and the inference rules, then the set of all formulas that can be deduced from combinations of the formulas earlier listed and inference rules and so on.

You can create a program P such that given a formula f as its input, it traverses the enumeration of all theorems and stops when it finds f (and, if f is not a theorem, it never stops as the enumeration is infinite). If we could decide whether P, given a certain formula, will halt, we would be able to know if a given formula is a theorem or not.

Unfortunately, we are not.


See also: GeneralHaltingProblemProblem



It is false to consider the HaltingProblem as a major thorn in the side of those who would like to be able to use mathematics to prove whether or not a program is correct. This is done. Just like it is shown for many programs whether they halt. It is just impossible to generalize this to an algorithm or program which works on all programs.

But wait a minute - doesn't this imply a constant amount of time for each instruction the machine executes? If we could build a computer that executes each instruction in half the time of the previous instruction, each instruction still will have a non-zero execution time, but it would be possible to execute an unlimited number of instructions in limited time. -- MW If (something false) then (anything you want to do can be true). Also, "time" is measured in terms of "clock cycles", not "clock cycles per second."

I am reminded of TheInfinityMachine?, see http://www.chiark.greenend.org.uk/~sgtatham/infinity.html -- MatthewAstley

It has been speculated that the physical conditions in the universe under gravitational collapse could enable computation in this manner (exponentially decreasing time needed per computation), due to the contraction in space and increase in energy density. See http://www.math.tulane.edu/~tipler/summary.html


From: http://www.math.hawaii.edu/~dale/godel/godel.html#HaltingProblem, a page about GoedelsIncompletenessTheorem:

HaltingProblem Theorem. There is no program R(p,i) which for each program p and each input i, can determine "yes" or "no" if p halts on i.

Proof.

See also: http://www.cs.washington.edu/homes/morrison/598/computability.html


One should note that the proof above applies only to a general purpose halting prover. If the program to be decided (p) is restricted to be smaller than the prover (R), then R(R, <any input>) is not allowed. Note that the size of the prover (R) could be much larger than the largest program allowed (p).

For instance, consider a limited prover which determines whether any 16-byte program in 6502 assembly language will halt given less than 8 bytes of input i. It is conceivable that a small problem like this could be solved using a prover R which is much larger than 16 bytes. Indeed, one could imagine a 4-gigabyte prover program which could decide the halting problem for all 1024-byte sequences of 80386 code with up to 512 bytes of input. (I'm not volunteering to write it.)

Why not just use a big lookup table?
Because there are 2^8192 different 1024-byte sequences (if we agree that a byte consists of 8 bits). Considering that there are only about 10^80 atoms in the universe, it is just impossible to save all byte sequences, even if every atom in the universe could hold a googol bits. -- PhilipBusch

Another way around the problem is to create environments where programs always halt. (One could argue that some operating systems act this way in practice. :-) For instance, in a Java environment one could count bytecodes and halt after a certain number. The prover in this case is trivial (if the programs cannot alter the counter).

Still another approach would be to create a compiler which rejects programs unless they are proven to terminate. Such a compiler may reject otherwise legal programs. One would build programs by making legal transformations which always guarantee the program will halt. (One rule might prevent altering a pointer unless one could guarantee that it did not cause a circular data structure. Alternately, a possibly-circular structure could only be traversed using algorithms which terminate.)

In short, while it is not possible to write a completely general halting prover, it may be possible to write a prover for any finite interesting program. I suspect that RealTime theorists are still working on the related problem of ensuring that certain instruction sequences will complete within a certain time period. -- OpenAuthor Er, so the prover itself wouldn't qualify as a "finite interesting program"...?


Great clarification, thank you. But does this lend any hope to the prove-the-program-correct people? Their cause looks pretty bleak to me.

Not so. It is possible to make a contribution without reaching a stated (or implied) goal. A quick web search for WeakestPrecondition? seems to indicate that that approach to reasoning is valued in the cryptographic community at least.

See also, EwDijkstra., A DisciplineOfProgramming, Prentice-Hall, 1976.


The fact that there are programs that cannot be proven to halt does not mean that people who want to prove programs correct are wrong. In general, the people who are trying to prove programs correct are not trying to take some random program and automatically prove things about it. Instead, they are constructing programs that they can prove things about. Programs should be understandable. If you can't prove that a program even halts then it is going to be hard to understand. Don't write programs like that!

The problem with proving program correctness is that it is too much effort for too little benefit. The benefit varies, of course. There have been a few times in my life where it was worthwhile to prove some properties about a program, and I did it. But most of the time, it is not worth the time involved. -- RalphJohnson


Here's another way to think about it:

Programs can be written that are equivalent to mathematical problems. For example, you could write a program that searches for a counter-example to Fermat's last theorem. If it finds one, it halts.

If you had a tool that solves the halting problem for any program, it would automatically be able to solve all sorts of hard mathematics problems with no work. It's not reasonable to expect such a magical tool to exist, so we shouldn't be surprised that it doesn't.

However does this say anything about the programs we actually write? Many of them aren't equivalent to tough mathematical problems. A program prover that works for many cases but sometimes says "I don't know" would still be quite useful. At least, it would focus our attention on the cases it can't prove.

-- BrianSlesinsky


It seems to me that it might be useful to write a program that took a time-limit and had three return values:

The third return value might also provide a means to continue the search (a continuation, in Scheme parlance).

Really I don't think the complexity of the halt-detector is a concern so much as the time or memory that it uses. Here's my halt detector:

Define such a thing as the "state" of the program under test, including the pointer that points to the next part of the program's input. Start the test-subject in its start state. Move the test-subject to its next state, repeatedly - but remember all the states the test-subject has ever been in. If the test-subject moves into a state it has been in before, then it will loop forever. If it terminates, then it will provably terminate. The concern is that the test-subject will just accumulate distinct states without ever terminating; then, your memory will run out! You can trade memory for time; you can, after all, calculate any state on demand, given the start state and the number of moves to take. But still, if the program can keep coming up with distinct states, it can out-run the halt detector.

A human would be able to detect patterns in the states and perform induction upon them. What's hard is figuring out how to get the halt-detector to do that. Even if it does do it, it still falls prey to the original problem: somebody who knows what sequence patterns it will detect can make a sequence which follows a pattern it won't detect. Feeding the halt-detector a version of itself guarantees that this will occur.

-- EdwardKiser

It isn't nescessary to know how to find a pattern, it's existence is enough. If such a pattern exists it is possible to create a proof enumerator which enumerates all possible proofs until it finds the right one describing te pattern and the induction upon it. It'll take some (lots of) time, but it is computable. -- AnonymousForPrivacyReasons

The previous solution gets considerably harder once you factor in the fact that the "verifier" should cope with more than a single input per program. What if it the program being verified could accept any length of input? By definition there is an infinite set of states. -- ArnonKlein

The way the problem has been classically defined, the particular input of the program is part of the question. In other words, we are not just asking if program X halts; we are asking if program X halts when given input Y. Whether the machine halts when given the same program but input Z is considered to be a separate question. -- EdwardKiser


There have been a few times in my life where it was worthwhile to prove some properties about a program, and I did it.

LoopInvariants? are a great help in writing all but the simplest loops.

Similarly, StateInvariants? are helpful in writing classes with non-trivial state. These can sometimes be checked at runtime using assertions to help in testing.

But the most useful proof technique is abstraction, i.e. specifying what is required separately from how it is implemented. Abstraction is crucial to good class and subsystem design. SystemMetaphors are often stated in terms of abstractions of the components of the system.

-- GlynNormington


But does this lend any hope to the prove-the-program-correct people? Their cause looks pretty bleak to me.

This is a common misunderstanding of the HaltingProblem Theorem. The HPT does not mean that, given a particular program, it is impossible to prove that the program halts. Proving that a particular program halts can be arbitrarily difficult, but for any program that is understandable by a human being, it is usually trivial.

So there is actually no theoretical problem with the idea of proving programs correct. As a matter of fact, people are doing these things as we speak.

-- StephanHouben

Besides, a perfectly correct program needn't necessarily halt; consider a program that finds solutions for FermatsLastTheorem or simply think of the implementation of an endless loop. Both can be correct, but won't halt if they are. -- PhilipBusch

"Proving that a particular program halts can be arbitrarily difficult, but for any program that is understandable by a human being, it is usually trivial."


The real question with proving programs correct is not: Can I prove this program correct? But: Can I build all interesting and useful programs using only two things:

It's actually fairly easy to set up systems using these two components; however, the programming exercise becomes a constant battle with the machine, trying to figure out how to express your ideas using only those constructs the machine will find acceptable. Oh, and don't expect to be able to feed this thing your employee payroll database as input if you have any kind of complexity going on...


Yes, the HaltingProblem is often misunderstood: Some years ago, I was working on a case tool project in the R&D division of a large multi-national company. They claimed that their case tool, and all the code it produced was "provably correct" because they added a counter to every loop, ensuring that it would terminate. Instead of...

  while (not eof(file)) {
read;
process;
  }
They'd do this:
  counter = 65535
  while (not eof(file) and counter-- > 0) {
read;
process;
  }
As you can see, proving that their programs terminated was a trivial exercise: All loops had a fixed maximum number of iterations. (And they did no recursion.)

However, in fact, their programs often crashed, often produced incorrect results, and the code they generated often didn't work.

I can prove that my Windows OS terminates. But I don't think that the BlueScreenOfDeath is a good thing. ;-> -- JeffGrigg


A rarely thought about consequence of the HaltingProblem is that if you have a LittleLanguage that you really want to optimize well, it may be worthwhile to keep it from being TuringComplete. For instance a RelationalDatabase is supposed to analyze and optimize queries, so SQL is not TuringComplete.

-- BenTilly

Why can't you implement a Turing machine in SQL??

Because any possible SQL statement can be implemented by a program that you can easily prove halts. This is because there is no construct in SQL with which you can implement a general loop. -- BenTilly

Are you sure? I'm no SQL expert, and I know SQL has no explicit looping mechanism, but how is it that you know some perverse and impractical combination of other SQL constructs and tables cannot be made to simulate a loop? I can believe that the relational algebra is not TuringComplete (I've read theoreticians who ought to know claim as much), but SQL gives you more than the relational algebra. For example, it seems I could model a "general loop" by writing a query that reads rows from some table T, and then adds new rows to the end of that same table T. Plus, if you look at the mathematics of relations, lots of non-trivial computations can be easily modelled in SQL, such as finite arithmetic, logical satisfiability, general constraint satisfaction, etc. SQL may well be Turing-incomplete, but to me, at least, it's far from obvious.

I am sure. The execution of any select statement can be implemented by constructing the set of all combinations of all tables mentioned in the FROM clause, removing those rows that do not fit the conditions specified in the WHERE clause (if present), applying the grouping rules specified in the GROUP BY clause (if present), filtering again by the conditions specified in the HAVING clause (if present), sorting this output according to the ORDER BY clause (if present), and then displaying results as specified in the SELECT clause. This is a program that executes in polynomial space and time, where the polynomial has the degree of the number of tables specified either directly in the FROM clause or mentioned in any way in any subquery.

Every other basic type of SQL statement also can be implemented naively in polynomial. time. (Ideally, of course, your database finds a more efficient representation.)

Indeed. In fact, any computer program could be viewed as a series of constant-time bit-setting operations. I don't see how this relates to the halting problem. There are plenty of programs - and SQL queries - that cannot run in polynomial with respect to the size of their input.
  • Nope! The important thing about computer programs is not bit-setting in variables, it's that they can do arbitrary (non-deterministic, potentially infinite) looping and/or recursion. SQL allows neither. It is quite trivial to prove that SQL allows neither. You can't write an infinite loop in SQL no matter what you do. It is also the case that you can't have TuringEquivalence? without such. As a comparatively minor side-issue, your assertion that SQL queries cannot run in polynomial time of their input is misleading to the point of being incorrect - they always run in polynomial time of their input, when input is defined to be both the query plus the database size. You presumably are thinking only of the query as input, but that is an erroneous approach; the result of such analysis is not helpful, whereas including database size as part of the input does yield a useful analysis.

Define the size of the input. My definition is the amount of data in the tables being processed. (The length of the query is, of course, the length of the program. Which has nothing to do with its algorithmic scaling.) If you disbelieve that with this definition of input there are plenty of SQL queries that cannot be run in polynomial time, then I would be interested to see a specific example. -- BenTilly

This means that it not only halts, but is guaranteed to do so in polynomial time.

No. It doesn't follow that because all of the operations of a language are polynomial time that all algorithms in that language halt. It doesn't even follow that all "programs" in that all halting programs do so in polynomial time with respect to their input size (which is the standard definition of polynomial and exponential time). Consider this loop:

while 1 == 1:
pass

All the operations in this loop run in polynomial time, so by your argument, this halts in polynomial time. How can you be sure that some cheap trick like this can't be played in SQL to create a non-terminating query? Why can't I make a query on some table T that contains a sub-query that, say, keeps appending rows to T, so that the enclosing query never hits the end of T?

You cannot make that query because there is no way to specify it in the SQL language. You cannot have a query that recursively refers to itself. Queries may have subqueries, and subqueries their own subqueries, but there is only a finite amount of nesting.

What about "recursive WITH" in SQL-99? (used for parts-explosion and similar "transitive closure" queries)

More specifically your example does not follow my argument. My argument is that any given query has a formal description of how to execute it where you construct a polynomial amount of data (the set of combinations of all tables in the queries), and pass this data through filters that are at worst polynomial per row, grouping and sorting operations that are n*log(n), other filters that are at worst polynomial, and a display operation. Since this comes down to doing polynomial things on a polynomial amount of data, the result is polynomial.

A full proof would have to proceed by strong induction on the depth of nesting of subqueries. Any SQL select statement without subqueries can be executed by taking a polynomial amount of data (size the product of the FROM tables), passing it through a series of constant-time filtering operations (as specified in the WHERE), then possibly send the whole through an n*log(n) grouping statement (if GROUP BY is present), possibly followed by constant-time per output row filtering (specified with HAVING), followed by an n*log(n) sorting operation (ORDER BY), and then the display operation (SELECT).

After that you can demonstrate that if every depth n or less SQL query can be executed in time that is polynomial of degree the number of tables times some sub-linear factor (from the n*log(n) operations), then so must be every depth n+1 SQL query. Which proceeds as before except that various operations that were previously constant-time might involve subqueries that execute in polynomial time.

But I've written an infinite loop as a single SQL statement. --BottomMind

Another well-known LittleLanguage that is not TuringComplete is any type of RegularExpression. First of all any true regular expression can execute in time proportional to the length of the string times the length of the regular expression with the right DFA engine. (Some DFA engines execute in linear time but possibly have exponential compilation time. Others finish in quadratic time.) Most things that we are used to calling regular expressions have irregular extensions and those engines are NP complete - but still are guaranteed to finish in exponential time.

-- BenTilly


I believe most common operating systems already support this. Run the program. Look at the task scheduler in a little while to see if it has halted. Don't waste time trying to analyze the program, just run it and see what happens.

Most people intuitively think of this solution when they first encounter the Halting Problem. But like the other 'solutions' to the HaltingProblem, it only works for a subset of all the possible programs. In this case, an easy counter-example would be a simple program which waits for 60 minutes, then halts.


"Gödel's Incompleteness Theorem And Turing's Solution To The Halting Problem: Linking The Foundations Of Mathematics And Computer Science" by Philip Rash http://www.philip.rash.com/research.html (mirror: http://web.archive.org/web/20010519155036/www.philip.rash.com/research.html)


Would you actually ever hit the paradox in the halting problem? It seems to me like it would end up infinitely calling itself before starting the evaluation. (Which might be detectable by the fictitious check_for_halt program).


I agree. It does seem to me that the example programs given as "proof" either infinitely repeat the calling up of a program or end with a program trying to execute with no input.

Does trouble(t) halt? (where string t represents the algorithm trouble)

function trouble(string s)

     if halt(s, s) = false
         return true
     else
         loop forever
Doesn't trouble(t) just infinitely repeat "if halt(t,t); if halt(t,t)...?"

Question: Can anyone write a simple program (that would actually run) as an example of the halting problem, that doesn't just boil down to the illogical statement: "IF P(Q)=no, then P(Q)=yes", or a quest for a mathematical conjecture?

The "proof" of the halting problem given in section 2 of this page has a problem: "program P that can tell you whether any program halts or not for given input data." and "program Q based on P which, if P says its input program doesn't halt, immediately halts, and if P says the program halts, goes into an infinite loop.* Feeding Q(Q) to P..."

When you feed Q(Q) to P, P begins to execute Q(Q), which makes Q begin to execute P(Q), which makes P begin to execute Q, which makes P begin to execute with no input.

It seems to me that all you need to do is add one more line of code to program P that reads something like "If P has no input, output=ILLOGICAL PROGRAM, halt.

-- DavidMonroe (yes, I'm an amateur programmer, but expert toxicologist at the US EPA)


See WhatDoesHaltingMean


Part 2 : GeneralHaltingProblemProblem


[[What follows below is simply misunderstanding about the Halting Problem and the proof that it is undecidable. Instead of P and Q, below, let me use Halt for the alleged program (of one argument) that solves the Halting Problem, and Spoiler the program (of zero arguments) that shows that Halt cannot work. It is a plain and simple result that Spoiler will cause Halt to fail. That is unavoidable. If you want, you can take the text of Spoiler, and incorporate it into another program Halt1, which tests to see if its input is the program Spoiler. But that program Halt1 will have its own Spoiler1. There is, quite simply, no way to avoid this. It simply does not matter what sort of examination Halt makes of its input tape, or whatever. Spoiler takes Halt as given, and is constructed to make Halt fail. That's it. End of story.

Nor are the attempted objections to the misunderstanding relevant. Even if Spoiler includes the text of Halt directly in some sense, Halt will not be able to exploit that fact. Spoiler is constructed by definition to make Halt fail, the proof that it does so is simple. There is no need to invoke an "obfuscated" version of Halt inside a virtual machine to thwart it. In fact, the problem of whether or not an obfuscated version of a program is equivalent to the original version is also undecidable (it is the problem of program equivalence), but this fact is proven by reducing the halting problem to the problem of program equivalence, not vice versa.

But ... there is apparently an even more basic misunderstanding which has to be addressed, because of the potential mechanics of Turing Machines and how they might be put together

It was to clear up this misunderstanding that the argument about an "interpreter" was invoked below. Let's summarize it up here. It relies specifically on Turing Machine formulations of the problem. Halt is defined so that, on a tape that contains nothing but a description of a 0-argument Turing Machine, it runs and leaves a tape which contains only a 0 or a 1, depending on whether or not the given Turing Machine halts on an empty tape. (This is all without loss of generality, I think everyone can agree.). The alleged refutation relies on the fact that when Halt is run, it need not assume that the tape is empty except for its own input. The tape may contain some other markings that a containing program, such as Spoiler, has placed on the tape. It is alleged that by examining these markings, Halt can alter its output to get around the problem Spoiler presents it with. But this is quite simply false. This is where the "interpreter" comes into play (not to obfuscate, as I thought was being suggested, but) by preventing the version of Halt contained within it from accessing anything outside what it would be able to access had it been started under normal conditions. In fact, it is not Halt that is contained within Spoiler, but a lobotomized version of it that forces it to work only on the Halting Problem and not anything else weird involving going outside its own input. It may be objected that this is not fair, restricting Halt in this way, but of course it is totally fair, because there is no obligation to include Halt inside Spoiler. All that is required for the simple proof to go through is a program that is input-output equivalent to Halt when it is started on as tape containing nothing but the program whose halting status it is supposed to ascertain. An "interpreter" of some form may be required to do this (such as one that simulates that otherwise empty tape, even while the program doing the simulation writes other stuff on the "real" tape). But the key is that the modified Halt, contained as a subroutine in Spoiler, is input-output equivalent to the original Halt on the actual problem that was given to the original Halt. It is irrelevant that it is not input-output equivalent on some other strange problem.]]

[[Uh, no. There is no need to be concerned with the undecidability of program equivalence here. There is no need for "uncrackable" anything, whatever that means, and certainly the version of Halt used by Spoiler (Halt') is more than "any source that expresses the same function" as Halt (how would one find such a thing if not as a provably equivalent minor modification of Halt itself?) It is in fact essentially Halt, but Halt restricted to run on a tape that it was required to be designed for. Note that Halt must be able to run on a tape that starts with nothing on it but a representation of the program it is required to test. It may contain some extra code to look elsewhere on that tape in order to see if it is "being tricked", however, it must be able to solve the Halting Problem without applying that code, in the circumstance that it is started normally on a tape containing only its own input. It is thus a simple matter for Spoiler to take the tuples defining Halt, and alter them so in effect they are running on a virtual tape/head combination, instead of the real tape/head (which is simulating them), and, most importantly, deliver the same answer as would be delivered by Halt running on the real tape/head combination, which, by the claim of its author, has input/output behaviour defining the Halting Problem, and which is the minimum anyone claiming to solve the Halting Problem must be able to provide. Given this using of the simulated Halt, which has input/output behavior equivalent to the original Halt, it is the usual simple proof to show that Halt applied to Spoiler cannot deliver a consistent result.

You know, computability theory is a highly developed and very interesting discipline. Do you really think you have found some problem at the very basis of it that renders all of the deep and complex results (such as those describing the highly complex structure of the recursively enumerable degrees) somehow flawed? Perhaps you should have a little less hubris. I see this has all been explained before, below, although perhaps the explanation that stays at a more concrete level (keeping always in mind that we are dealing with concrete Turing Machines, not lisp machines, abstract environments and interpreters not directly reduced to Turing Machines, etc) may be more helpful. But maybe nothing is helpful for a troll?]]

Brand new semi-humorous argument: my machine P is both a GeneralHaltingProblem solver and also good at inferring the intent of its input. When invoked as Q's inner loop, P will examine its input (source of Q) and immediately realize that it (P) is being used in a misguided attempt to prove that it (P) cannot actually exist. It (P as Q's inner loop) will consequently print "0" - causing Q to halt - while simultaneously adding states to its (Q's) FSM which, when inspected by the outer invocation (P), explain the subterfuge, permitting it - the outer invocation of P - to assert the correct halting value. Which is, correctly, "1".

Please email the Nobel to Pete@that-place-I've-gone-to.com. Thanks!

But seriously, where's the logic error above?

You're brilliant, Pete, but you just overstepped the limitation of Turing Machines. Programs for TM are deterministic, and therefore two runs of the same program for the same input cannot give different results.

If we allow for non-determinism, then, yes, you can make a program P that somehow (magically) distinguishes when it is run from the supervision of Q and does whatever trick it needs. But then, you haven't disproved the HaltingTheorem because such a program cannot be constructed in TMs. So, you don't get the Nobel.

 ;assume we have a program p
 ;without side-effects, in Scheme
 (define p (lambda (x) <body-of-p>))

; and p solves the halting question for a program x returning ; true if (x) halts and false otherwise.

; then we define the program q (define (q) ( if (p q) (loop-forever) #t)) ; and if p doesn't have side-effects (it is purely functional) ; then (p q) should yield the same result in all context, ; and it is easy to see that ; (p q) = true implies q loops forever

; Now the inventive Peter, will protest that q has access to an environment ;(which makes the recursive invocation (p q) possible, so then if p has access to the ; environment than p can distinguish the places of invocation ; therefore the only technical trick to be done is to de-recursify q, ; remove all access to the environment while keeping it a program equivalent with the above, ; To do that we use a fixed-point combinator like the U ; http://okmij.org/ftp/Computation/fixed-point-combinators.html (define U (lambda (f) (f f)))

; we can then define q as (define q (U (lambda (f) (if (p f) ((lambda (x) (x x))(lambda (x) (x x)));loop forever #t)))) ; now we eliminated the recursion while keeping q the same ; let's inline the lambda expression for U and p ; let's inline the lambda expression for U and p (define q ((lambda (f) (f f)) (lambda (f) ( if ( (lambda (x) <body-of-p>) ;p actually, applied to (f f)) ; applied to (f f), which is actually a stand-in for q, ; then ((lambda (x) (x x))(lambda (x) (x x)));loop forever ; else #t)))) ;therefore if <body-of-p> exists with the above property, ;there exists the lambda expression denoted by q ;and evaluating ((lambda (x) <body-of-p>) ; p applied to ; q unrolled ((lambda (f) (f f)) (lambda (f) ( if ((lambda (x) <body-of-p>) (f f)) ((lambda (x) (x x))(lambda (x) (x x))) #t)))) ;we reach the conclusion that body-of-p cannot ;exist with the specified properties


 (define q (if (scheme_interpreter '(p q))
                  (loop-forever)
                   #t))



In any case, you guys can quit your crowing on the A-machine. Costin is, above, assuming the existence of an interpreter that can't be cracked. Let's call it I. If you'll kindly refer me to a proof that such an interpreter can exist, then you're out of the woods. To forestall the usual squawking, no, I don't know that you can't prove such a thing - but the burden of proof remains with you. It sure doesn't smell recursive ... -- Pete

Ahem, I beg to differ, Pete. If we restrict the discussion to a purely functional subset of Scheme, then such interpreters of a functional subset of Scheme are written by students around the globe all the time. That they cannot be "broken" follows trivially from the semantic of Scheme: if all the primitives (car,cdr, define, let, if` etc) are deterministic, and all the composition constructs are deterministic, it follows by structural induction that all constructed procedures are deterministic so that evaluating the same expression in any correctly implemented interpreter will always yield the same result.

In principle, of course, I agree. Or else I would have popped up a counter-example. But a specification is not a proof; you'll need to start by proving that each of these primitives cannot be cracked, then show that their principal of combination cannot be cracked. Given RicesTheorem (http://mathworld.wolfram.com/RicesTheorem.html) this is not a trivial undertaking. At each step you risk the AntecedentAssumedFallacy. This is why I say the uncrackable interpreter smells non-recursive.

Mapping from a purely functional subset of scheme to lambda calculus is also a beaten path, so if you want a full-blown interpreter to be presented to you as part of the proof, you'd better give us some good reason rather than asserting the burden of proof on the other side. Because it's surely boring to write down that interpreter and it provides no insight into the HaltingProblem whatsoever. A quick reference could be SiCp, but examples of such interpreters are so many, I don't know where to begin.

I don't actually expect you do any such thing. It sounds like a frightful waste of time. But until you or someone does, the A-machine version of the halting problem remains cracked. And then as Doug points out most likely TM != LC.


There's a valid conclusion to be drawn that maybe TM are not as good as real computers (those can do all kinds of non-deterministic tricks, like being connected through networks, running MS Windows, or using non-ECC memory :) ), but then the stuff that we need TM for can be adequately modeled using TM.

Even more serious than CC's critique: there's no Nobel prize in math. There's the Field's medal, but you may be a bit long in the tooth for that.

Yes, apparently a mathematician raced off Nobel's wife. Strange thing, science ;-)

Keep in mind that Turing's original machines are far from the only model of computation ever studied. Machines with oracles are non-deterministic, for instance, and there are also randomized machines, which do indeed have different performance. For instance, the computational complexity of some algorithms depends on whether the Extended Riemann Hypothesis is true, which no one knows is true or not - however, such algorithms can typically be converted into the faster computational class simply by using randomness, which doesn't exist in the original Turing machines. It's a pretty big topic, and TuringMachines are simply a starting place. -- Doug


The most fundamental plank of ComputerScience is the General HaltingProblem. Unfortunately there appears to be a problem with the GHP, as follows:

Therefore,

I don't have any such program P and anyone else who says they do is full of it.

But:

Realizing this, it's plain that the whole form of the proof is identical with the convict's invalid reasoning in the UnexpectedExecutionParadox. All the computability results that rest on the GHP are therefore null and void, and we better start over on this topic. Oh, another Nobel for me? Pesky things, I keep tripping over 'em. -- PeterMerel


I can't quite tell whether this is a joke or not. Assuming it's serious (DeleteMe if it's just a joke): There is no fallacy. Here's one way to adjust the proof that may make it clearer that there is no fallacy.

Hence, we've shown that no program P can be a halting tester in the precise sense described by that statement. And we haven't had to assume that any program is a halting tester in order to do it. We started with any candidate program, and produced a new program for which it doesn't give the right answer. -- GarethMcCaughan

I expected this semi-humorous argument to go down in flames, but couldn't quite put my finger on why just yet. Anyway you're quite right, many thanks. A more serious if exformal objection is raised on WhatDoesHaltingMean and discussed on GeneralHaltingProblemProblemProblem, suggesting that InfiniteLooping is in fact exactly what halting means. Ergo Q can not fail to halt, and the Turing fence is down again ... -- PeterMerel

The clearest technical book on the subject that I've ever seen is The Language of Machines - An introduction to Computability and Formal Languages by Floyd and Beigel, ISBN 0716782669 . Obviously, these things cannot be adequately covered non-technically/non-mathematically. -- DougMerritt


Here's a program (in an imaginary language, for an imaginary machine) that might explicate things.

 # ALLEGED_TESTER is a function which accepts a single argument.
 # We return a new function F that proves that ALLEGED_TESTER isn't
 # a perfect termination tester: in other words, either F terminates
 # and ALLEGED_TESTER(F) doesn't return 1, or else F doesn't terminate
 # and ALLEGED_TESTER(F) doesn't return 0.
 def thing_not_tested_validly(alleged_tester):
   def f():
     tester_result = alleged_tester(f)         # same f as we're now defining
     if tester_result == 1 then loop_forever()
     return
   return f

Proving the assertions in the comments there is easy. If ALLEGED_TESTER(F) returns 1 then F loops for ever. If ALLEGED_TESTER(F) returns something other than 1 then F doesn't loop for ever.

If you're bothered by the "same f as we're now defining" thing, don't be: there are ways of working round that. But this is one of the subtle points of the proof. -- GarethMcCaughan


What is wrong with the first proof? It's proof by reductio ad absurdum. To prove that P can't exist you assume P does exist and make it lead to a false conclusion, given your knowledge base.

If you do away with this kind of proof you do away not just with the GHP but with most of mathematics too. :-)

Nothing is wrong with it. I provided the second one because I thought it might evade some mental blocks. It might be worth mentioning that a similar thing can be done to many proofs by reductio ad absurdum, for the sake of those who find RAA hard to trust. -- GarethMcCaughan


Let me know if I get this:

Nice...

What this is really telling me is that there's no way to actually completely test a program except by running it. -- CristiOpris

Yes, that's the idea. It's a proof that actually we don't have a program P that tells whether things terminate or not. And it's even worse than you say :-), because even running a program can't prove that it doesn't terminate. -- GarethMcCaughan


Yes, the "general halting problem" proof is one that is commonly given in college courses. (At least it was in the early '80s. ;-)

But I think it's flawed.

Briefly, to know if Q halts, you need to know its input. Its input is Q, but to know if that Q halts, you need to know its input. So, you're really testing this:

  Q(Q(Q(Q(Q(Q(Q(Q(Q(Q(Q(Q(Q(Q(Q(Q(Q(Q(Q(Q(Q(Q(Q(Q( ... ))))))))))))))))))))))))
Where there's an infinite number of Qs.

The definition of the thing you wish to run doesn't halt, so you can never get to the point of running it. -- JeffGrigg

Ahhh, yes: THAT is the approach I was looking for. Thanks, Jeff! -- TomStambaugh

No, what you input to Q is the "source code" of Q, so no need to get all recursive here.

No, there's no way to know if a program will halt or not unless you know the input. Consider this program: { if input = "halt" then halt else loop_forever } Now, does it halt or run forever? You can't know if it halts unless you know if the input is "halt".

So, to evaluate if Q halts, you have to know its input halts. That input would be Q, so...

Yes, but the input to Q is not the result of executing Q, it's nothing at all. Here's a slightly reworded definition that makes that a little clearer:

-- GavinLambert

I think the problem might be relating to the definition of 'input'. I'd say that, although there is no explicit input to 'Q()' (according to the signature), there is an implicit input, in the form of the call to 'P' ('Q' asks for input from 'P'). With that call, 'P' ends up having to analyse itself with the associate input of 'Q'... and now we've got a problem.

As an aside, there would seem to be no reason that 'Q' couldn't detect the situation, that it has processed itself multiple times, or even processed itself at all, and therefore respond to the question with a question, so to speak: "Does 'Q' halt? I don't know... do you want it to halt?"

-- WilliamUnderwood

OK, so suppose we have this program "R" which can analyze an input and halts if the input is equivalent to itself...


"Does 'Q' halt? I don't know..." Well if 'P' doesn't know then it isn't solving the halting problem. If you ask the question "does this program halts?" and for some programs like 'Q' you are forced to answer something else than 'yes' or 'no', then Turing is right: you cannot write a program which can answer that question with a 'yes' or 'no' for any given input program. -- Samuel Gélineau


To break down what I see as the problem here, at least on the top part of the page, the proof is this: "Even if you had a halting predictor, you don't have a halting predictor. I can do this for all putative halting predicters." To put something else in that form, "Even if you think you have an integer that is greater then all others, I can prove that your integer isn't greater then all others (by simply adding 1 to yours). I can do this for all putative largest integers."

Your inability to produce the integer that is greater then all others isn't a problem. I never really depended on your specific integer, I made a statement about all of them, by showing concretely that each and every one of them is inadequate. -- JeremyBowers

Hmm...

Assuming that a program could be defined which can recognize itself as input:

From any halting detector, you can provide a program which halts or not based on the opposite of the decision of that detector.

And for any such program you provide me which does that, I can provide a program which can detect trickery of that specific nature.

We can't solve the halting problem in general, but we can get as close as we'd like.

Now, "Can this result be made useful?" is a completely different question.

-- WilliamUnderwood

P.S., where n = the biggest number you can quote to me, the number returned by (n+1) is the largest integer. Prove me otherwise :p (n+1)+1 - I don't think you really understand mathematical symbology. Anything goes in those variables, not just pure numbers.

It is well known that it is, on the other hand, feasible to solve the halting problem for all machines up to size K, for any given finite constant K you care to name. The machine that solves that particular finite K-halting problem will be bigger than K (O(2^K) does it nicely). Unsurprisingly, the class of machines it can solve does not, therefore, include itself. -- dm

Hence the answer of "I don't know. Find a bigger machine." :)

Also, yes, it is useful. Many proof techniques are based on exhaustive examination of state space for things that are equivalent to solving the halting problem. Since the state space is finite, it is sometimes feasible, whereas in the general case it clearly isn't. This is another critical thing to understand about the Halting Problem non-Problem: It specifically refers to the general case, and does not apply in less general cases. Though even in the less general cases it does tend to imply an irreducible exponential cost. -- JeremyBowers


Why need a "Halting Predictor" to prove the point...

How about "Is it possible to predict the output of the function f(int i){return i;}"? The function takes an int (let's say only 0 or 1) and spits it out. It is not possible to tell exactly what it spits out, whether a 0 or a 1 (without knowing the input). Doesn't this qualify as a "proof" that it is not possible to predict the output? If NO (it doesn't qualify), then how different is the "Halting Problem" from this one other than to disguise the input and output in some fancy programmer's formalism? -- Mohyneen Mehdi


You don't necessarily need something fancier, but you do need to consider all possibilities one way or another. You're basically assuming the result you want to prove; that's why this is not a proof.


Maybe I am missing something or maybe I am not, but I can't give in to the "circular reasoning" that lurks in the proof of the Halting Problem. In standard English, the solution goes like this... P has the task of predicting Q's output (or behavior). But Q has the final say in the matter and waits till P makes its prediction, after which Q conveniently adapts its behavior only to violate that prediction.

So, as I see it, P has to look at "Q AND Q's Input" in order to predict its behavior (or output). But Q's INPUT is P's OUTPUT. So, effectively, P has to look at "Q AND P's OUTPUT". But P's OUTPUT is obtained only by looking at "Q AND Q's Input". Back to square one! It's important to understand that by Q's INPUT I don't mean Q's parameter values; I mean information that Q eventually uses to behave the way it does.

P needs information which is obtained from Q, who in turn needs the "same" information from P. But according to the proof, that information is generated on the fly and P is given the credit (or discredit) for generating it. That information (i.e the prediction P makes) couldn't have been generated because P doesn't know Q's complete Input information, information which can come only AFTER P makes it's prediction (i.e the prediction itself).

Where exactly am I going wrong? -- Mohyneen Mehdi

Here's how I see it: Q's input doesn't have to be a precalculated boolean value equal to P's output. P is supposed to (among other things) look at its own source code and figure out for himself what P's output is supposed to be. If P wants to execute this code in order to find out then P will loop forever, won't ever give an answer for this particular program, and cannot solve the HaltingProblem in general. If P chooses to guess Q's input without executing its own source code, then whatever his guess will be he will give a wrong final answer about Q, so P still isn't solving the HaltingProblem in all cases. -- Samuel Gélineau


Where exactly am I going wrong?

There's several common spots of confusion. Maybe you can help us re-state or otherwise explain the problem so it's less confusing. (But I don't think there's any way of getting around the contradiction, since it's a proof-by-contradiction.) Confusion between what people mean by Q:

Other people are mislead into The same sorts of confusion have led many people to claim that QuinePrograms are impossible, when clearly several do exist.

Also, a few of the "proofs" of FreeWill seem to be based on the same confusion.

P needs information which is obtained from Q, who in turn needs the "same" information from P.

Yes, sort of. When P runs, it inspects the source code for Q, and also inspects the input that will be fed to Q.

Please note that P does not actually call() or eval() or execute Q. (Although the sorts of analysis it does are probably very similar to the sorts of analysis that go on when the compiler compiles and partially evaluates Q when we later compile Q).

But according to the proof, that information is generated on the fly and P is given the credit (or discredit) for generating it.

Yes, exactly.

That information (i.e the prediction P makes) couldn't have been generated because P doesn't know Q's complete Input information, information which can come only AFTER P makes it's prediction (i.e the prediction itself).

I think you're very close to understanding the HaltingProblem.

In this case, P *does* know Q's complete input information. Tell me the source code to P (long before you ever *compile* P), and I can tell you the long string that makes up Q's complete input information. It is, in fact, the source code to Q (which includes a copy of the source code to P that you gave me).

Say I want to know whether this function halts (i.e., whether it is an algorithm), and if so what value it returns:

  int count_apples(){
    int left = count_apples_on_left();
    int right = count_apples_on_right();
    return( left + right );
  }
By "input", we normally mean input from *outside* the computer. The value returned by count_apples_on_left() does *not* count as "input". I inspect that function and see that it calculates a value without any input from *outside* the computer. Say I run count_apples_on_left() once and it returns "17". I can be confident that it will return "17" every time I or anyone else runs that subroutine. Right?

So, what does count_apples() return? Or does it never halt?

I inspect count_apples_on_right(), and I see that it reads in 2 characters from standard input. Those 2 characters count as "input", not just to count_apples_on_right(), but to any program that calls it (such as count_apples() ), and to any program that indirectly calls it (perhaps by calling count_apples() ).

If I know what those 2 characters will be, do I have any hope of figuring out the output of count_apples()?


I just came to read here, correct me if I misunderstood something.

For halting property, there are three kind of program.

Halting problem states that (I took this from first line in HaltingProblem)

"It is impossible to write a program that runs in finite time that is able to decide whether an arbitrary program will halt (with a certain input)."

Let P be the halt tester program, and Q be the program that will be feed to P, and go in infinite if P return "Q will halt", and halt if P says "Q never halts"

Now the statement With a certain input is where the flawed is.

What is the input to first Q that is fed to P?

''The input is empty, or a dummy string, it does not matter at all. As a matter of fact you can consider the input to be part of the program, and that settles all the useless discussion about input. You cannot define a program P that decides for all programs if they halt with an empty input.The counter-example Q defined by

   (define (Q) (if (eval '(P Q)) (loopforever) true))
settles it. If you notice, Q does not use any input at all. So the input can be anything, it's better left out of the picture.

Technically Q is often constructed along the lines

  (define (Q X)  (if (P X X) (loopforever) true))
And the counter-example is that P cannot decide for
 (P Q Q)

Which is more faithful to the original formulation of the halting problem where P had to have 2 arguments (P X I) where X is the program and I is the input. But this later arrangement, while valid, makes it less intuitive, as the reader is distracted by the artifice of using Q both as program and input, and by the role of the dummy variable X in the body of Q. So reducing the problem to programs with empty input, makes the intuition much more accessible. So we prove that we cannot even construct a program that decides the termination of all programs with an empty input.

I took the liberty to delete the flawed discussion about the lookup tables, and the relative sizes of programs and lookup tables. It just distracts from a technically trivial proof. -- CostinCozianu

I don't see why you did so. I said explicitly that it was the starting place for one of the classic approaches to proofs, which I was trying to lead up to one step at a time - maybe not one of the proofs that you prefer, but I don't see why that's reason to delete it. It's possible that your explanation will be better understood by the current audience, but even that is hard to say - obviously, many people do not find the topic clear even when I think it's clearly presented (and yes, I think your statements are clear). I mean, I do see your point that the topic should be simpler, the way you presented it; the question is, will your audience get it. If they do, cool. -- DougMerritt''

I just reread the above and I see specifically what was subconsciously bothering me. I do not think you were clear in the critical part, the conclusion: "And the counter-example is that P cannot decide for (P Q Q)". I think you need to do something to make that much more glaringly obvious, or else your "simplified" approach won't get the point across. -- Doug

But that was exactly the point: that if we discuss about a program/procedure say P2 that takes 2 parameters: (P2 X I), we make it far less obvious to come up with (Q2 X) that evals (P2 X X) and turns the result on its head. Instead if we focus just on P1 that takes input a program that runs with an empty (or otherwise fixed) input then we construct Q1 that evaluates (P1 Q1) and turns it on its head. So assuming a purely functional subset of Scheme, the proof is:

 ;Proof 1
 ;assume there is a P1 that decide termination for procedures with no parameters
  (define (P1 X) (...))
 ;where forall X, (P1 X) evaluates true if (X) halts, false otherwise [1]
 ;construct the function with no parameter
  (define (Q1) (if (eval '(P1 Q1)) (loopforever) true)) ;[QDEF]
 ; then
 ;  if (P1 Q1) evaluates to true, it follows according to [QDEF] that (Q1) loops forever , contradicting [1] --  -- {step A}
 ;  if (P1 Q1) evaluates to false, it follows according to the definition of Q1 that (Q1) terminates , contradicting [1] -- [step B]
 ; therefore because (P1 Q1) cannot evaluate to either true or false without contradicting [1]
 ; the assumption that there exists P1 leads to a contradiction and is therefore false (reductio ad absurdum)

This is the complete sketch of the proof about the non-existence of a Scheme procedure P1 that can decide for all Scheme procedures with no argument if they terminate or not. Using code in a real language makes it easier to follow, while keeping it reasonably formal. If on the other hand we try to prove the version with (P2 X I), coming up with [QDEF] and verifying [step A] and [step B] is less intuitive:

 ;Proof 2
 ;assume there is a P2 that decide termination for procedures with one parameter ("input")
  (define (P2 X I) (...))
 ;where forall X, (P2 X I) evaluates true if (X I) halts, false otherwise [1]
 ;construct the procedure with one parameter
  (define (Q2 R) (if (eval '(P2 R R)) (loopforever) true)) ;[QDEF]
 ; then
 ;  if (P2 Q2 Q2) evaluates to true, it follows according to [QDEF] that (Q2 Q2) loops forever, contradicting [1] -- [step A]
 ;  if (P2 Q2 Q2) evaluates to false, it follows according to the definition of Q2 that (Q2 Q2) terminates, contradicting [1] -- [step B]
 ; therefore because (P2 Q2 Q2) cannot evaluate to either true or false without contradicting [1]
 ; the assumption that there exists P1 leads to a contradiction and is therefore false (reductio ad absurdum)

So I think that it is clear that Proof 2 is both harder to follow and harder to remember at a later time than Proof 1, and this responds directly top the anonymous contributor's question as to what happened with the input for Q. There is no input, and I presented the justification why it is better to look at the version with no input.

Now for GoldPlating we can tighten some details by to replacing recursion with a fixed-point combinator to make the proof adequate for the halting problem within LambdaCalculus {there's no lambda expression P so that for any other lambda expression X the application (P X) reduces to "true" (aka (lambda (x y) x) ) if X has normal form, and to "false" if X doesn't have a normal form }. Because LC and TM are known to be computationally equivalent this is as good as a proof of the halting problem for TM.


Part 3 GeneralHaltingProblemProblemProblem


There is no spoon. Intuitionistically speaking, all programs that cannot be inductively proven to terminate are illegal. For reactive systems, use coinduction proofs and replace termination with progress. Are there any interesting programs left?


EditText of this page (last edited November 26, 2014) or FindPage with title or text search